"Lagrangian formulation of electromagetism"의 두 판 사이의 차이

수학노트
둘러보기로 가기 검색하러 가기
imported>Pythagoras0
imported>Pythagoras0
2번째 줄: 2번째 줄:
 
*  Lagrangian for a charged particle in an electromagnetic field <math>L=T-V</math>
 
*  Lagrangian for a charged particle in an electromagnetic field <math>L=T-V</math>
 
:<math>L(q,\dot{q})=m||\dot{q}||-e\phi+eA_{i}\dot{q}^{i}</math>
 
:<math>L(q,\dot{q})=m||\dot{q}||-e\phi+eA_{i}\dot{q}^{i}</math>
*  action
 
:<math>S=-\frac{1}{4}\int F^{\alpha\beta}F_{\alpha\beta}\,d^{4}x</math>
 
 
*  Euler-Lagrange equations
 
*  Euler-Lagrange equations
 
:<math>p_{i}=\frac{\partial{L}}{\partial{\dot{q}^{i}}}=m\frac{\dot{q}_{i}}{||\dot{q}_{i}||}+eA_{i}=mv_{i}+eA_{i}</math>
 
:<math>p_{i}=\frac{\partial{L}}{\partial{\dot{q}^{i}}}=m\frac{\dot{q}_{i}}{||\dot{q}_{i}||}+eA_{i}=mv_{i}+eA_{i}</math>
15번째 줄: 13번째 줄:
  
 
==Lagrangian for electromagnetic tensor==
 
==Lagrangian for electromagnetic tensor==
 +
===free===
 
* 상호작용이 없는 전자기장의 라그랑지안은 다음과 같다
 
* 상호작용이 없는 전자기장의 라그랑지안은 다음과 같다
 
$$\mathcal{L}_{\text{EM}}= - \frac{1}{4}F_{\mu\nu}F^{\mu\nu}=\frac{1}{2}(\mathbf{E}^2-\mathbf{B}^2)$$
 
$$\mathcal{L}_{\text{EM}}= - \frac{1}{4}F_{\mu\nu}F^{\mu\nu}=\frac{1}{2}(\mathbf{E}^2-\mathbf{B}^2)$$
 
이 때 <math>F_{\mu\nu} = \partial_\mu A_\nu - \partial_\nu A_\mu \,\!</math>는 전자기텐서, $A=(A_{\mu})$는 전자기 포텐셜
 
이 때 <math>F_{\mu\nu} = \partial_\mu A_\nu - \partial_\nu A_\mu \,\!</math>는 전자기텐서, $A=(A_{\mu})$는 전자기 포텐셜
 +
*  action
 +
:<math>S=-\frac{1}{4}\int F^{\alpha\beta}F_{\alpha\beta}\,d^{4}x</math>
 
* 라그랑지안은 전자기 포텐셜의 다음과 같은 변환에 대하여 불변이다
 
* 라그랑지안은 전자기 포텐셜의 다음과 같은 변환에 대하여 불변이다
 
:<math>A_{\mu}(x) \to A_{\mu}(x)-\partial_{\mu}\Lambda(x)</math>
 
:<math>A_{\mu}(x) \to A_{\mu}(x)-\partial_{\mu}\Lambda(x)</math>
26번째 줄: 27번째 줄:
 
$$
 
$$
 
* see http://www.damtp.cam.ac.uk/user/tong/qft/six.pdf
 
* see http://www.damtp.cam.ac.uk/user/tong/qft/six.pdf
 +
 +
===in the presence of $j$ and $\rho$===
 +
* action
 +
$$S[\phi,A]=\int_{t_1}^{t_2}\int_{\mathbb{R}^3}-\rho\phi+j\cdot A+\frac{\epsilon_0}{2}E^2-\frac{1}{2\mu_0}B^2\,dV\,dt$$
 +
* w.r.t $\phi$
 +
$$\nabla\cdot E=\frac{\rho}{\epsilon_0}$$
 +
* w.r.t $A$
 +
$$\nabla\times B=\mu_0j+\epsilon_0\mu_0\frac{\partial E}{\partial t}$$
 +
 +
  
  
42번째 줄: 53번째 줄:
 
* [http://www.physics.sfsu.edu/~lea/courses/grad/fldlagr.PDF The field Lagrangian]
 
* [http://www.physics.sfsu.edu/~lea/courses/grad/fldlagr.PDF The field Lagrangian]
 
* http://www.lecture-notes.co.uk/susskind/classical-mechanics/lecture-8/the-electromagnetic-lagrangian/
 
* http://www.lecture-notes.co.uk/susskind/classical-mechanics/lecture-8/the-electromagnetic-lagrangian/
* http://unapologetic.wordpress.com/2012/07/16/the-higgs-mechanism-part-1-lagrangians/
+
 
 +
 
 +
==blogs==
 +
* Higgs mechanism
 +
** http://unapologetic.wordpress.com/2012/07/16/the-higgs-mechanism-part-1-lagrangians/
 +
** http://unapologetic.wordpress.com/2012/07/17/the-higgs-mechanism-part-2-examples-of-lagrangian-field-equations/
 +
** http://unapologetic.wordpress.com/2012/07/18/the-higgs-mechanism-part-3-gauge-symmetries/
 +
** http://unapologetic.wordpress.com/2012/07/19/the-higgs-mechanism-part-4-symmetry-breaking/
 +
 
  
  
 
==questions==
 
==questions==
 
* http://physics.stackexchange.com/questions/3005/derivation-of-maxwells-equations-from-field-tensor-lagrangian?rq=1
 
* http://physics.stackexchange.com/questions/3005/derivation-of-maxwells-equations-from-field-tensor-lagrangian?rq=1

2013년 3월 24일 (일) 16:39 판

Lagrangian for a particle

  • Lagrangian for a charged particle in an electromagnetic field \(L=T-V\)

\[L(q,\dot{q})=m||\dot{q}||-e\phi+eA_{i}\dot{q}^{i}\]

  • Euler-Lagrange equations

\[p_{i}=\frac{\partial{L}}{\partial{\dot{q}^{i}}}=m\frac{\dot{q}_{i}}{||\dot{q}_{i}||}+eA_{i}=mv_{i}+eA_{i}\] $$ F_{i}=\frac{\partial{L}}{\partial{q^{i}}}=\frac{\partial}{\partial{{q}^{i}}}(eA_{j}\dot{q}^{j})=e\frac{\partial{A_{j}}}{\partial{q}^{i}}\dot{q}^{j} $$

  • equation of motion\(\dot{p}=F\) Therefore we get

\[m\frac{dv_{i}}{dt}=eF_{ij}\dot{q}^{j}\]. This is what we call the Lorentz force law.

  • force on a particle is same as \(e\mathbf{E}+e\mathbf{v}\times \mathbf{B}\)


Lagrangian for electromagnetic tensor

free

  • 상호작용이 없는 전자기장의 라그랑지안은 다음과 같다

$$\mathcal{L}_{\text{EM}}= - \frac{1}{4}F_{\mu\nu}F^{\mu\nu}=\frac{1}{2}(\mathbf{E}^2-\mathbf{B}^2)$$ 이 때 \(F_{\mu\nu} = \partial_\mu A_\nu - \partial_\nu A_\mu \,\!\)는 전자기텐서, $A=(A_{\mu})$는 전자기 포텐셜

  • action

\[S=-\frac{1}{4}\int F^{\alpha\beta}F_{\alpha\beta}\,d^{4}x\]

  • 라그랑지안은 전자기 포텐셜의 다음과 같은 변환에 대하여 불변이다

\[A_{\mu}(x) \to A_{\mu}(x)-\partial_{\mu}\Lambda(x)\] 여기서 $\Lambda(x)$는 임의의 스칼라장

  • equation of motion

$$ 0 = \partial_\mu F^{\mu\nu} $$

in the presence of $j$ and $\rho$

  • action

$$S[\phi,A]=\int_{t_1}^{t_2}\int_{\mathbb{R}^3}-\rho\phi+j\cdot A+\frac{\epsilon_0}{2}E^2-\frac{1}{2\mu_0}B^2\,dV\,dt$$

  • w.r.t $\phi$

$$\nabla\cdot E=\frac{\rho}{\epsilon_0}$$

  • w.r.t $A$

$$\nabla\times B=\mu_0j+\epsilon_0\mu_0\frac{\partial E}{\partial t}$$



memo


related items


expositions


blogs


questions